Download as pdf or txt
Download as pdf or txt
You are on page 1of 53

Calculus of single variable

September 10, 2019

1 Function
Definition 1.1. A function f from a set X to a set Y is a procees that associates to
each element x of X a unique element f (x) of Y . We write
f : X → Y, x 7−→ f (x).
The set X is called the domain of f and the set Y is called co-domain of f . Range of f
is defined as
{y ∈ Y : y = f (x) for some x ∈ X}.
Definition 1.2. We call a function f injective or one to one if
x1 , x2 ∈ X, x1 6= x2 ⇒ f (x1 ) 6= f (x2 ).
Equivalently
x1 , x2 ∈ X, f (x1 ) = f (x2 ) ⇒ x1 = x2 .
Definition 1.3. We call a function f surjective or onto if
∀y ∈ Y, ∃x ∈ X such that y = f (x).
In other words, a function f is onto if its range is equal to its co-domain.
Definition 1.4. A function f is called a bijective function if it is both injective and
surjective.
Exercise: Construct a function f : N → N ∪ {0} which is bijective.
Exercise: Construct a function f : N → Z which is bijective.
Exercise: Construct a function f : (0, 1) → R which is bijective.
x− 1
(Hint: f (x) = x−x22 .)
Exercise: Construct a bijective function f : [a, b] → [c, d] such that f (x) is a linear
polynomial function and that f (a) = c, f (b) = d.

1
2 N, Z and Q
1. Natural numberss: N = {1, 2, 3, · · · }.

• Properties (Peano Axioms):


(a) 1 ∈ N.
(b) If n ∈ N then n + 1 belongs to N. We call n + 1 the successor of n.
(c) 1 is not successor of any element in N.
(d) If n and m have the same successor then n = m.
(e) A subset of N which contains 1, and which contains n + 1 whenever it
contains n, must equal N. (basis of mathematical inductuction)
• If we add two natural numbers, we get another natural number. Is the same
statement true for the substraction?
NO!!. (Perhaps we need to extend the number system).

2. Integers: Z = {· · · − 3, −2, −1, 0, 1, 2, 3, · · · }.

• We can add and substract two integers.


• Multiply.
• Can we divide?
NO!!. (Perhaps we need more numbers).

3. Rational numbers: Q = { pq : p, q ∈ Z, q 6= 0}.

• 1/2, −3/5, 1.333333 · · · ∈ Q.


• We can add and substract two rational numbers.
• Multiply.
• Divide.
• (Order Property:) We say a ≤ b if b − a is a positive number.
– Given a and b, either a ≤ b or b ≤ a.
– If a ≤ b and b ≤ a, then a = b.
– Transitivity: if a ≤ b and b ≤ c, then a ≤ c.
– If a ≤ b, then a + c ≤ b + c.
– If a ≤ b and 0 ≤ c then ac ≤ bc.

2
• Do we need more numbers?
• Consider a square with side having length 1. If d is the length of the diagonal
then by Pythagoras theorem, we have d2 = 2. So, we are trying to get a root
of the polynomial x2 − 2.
• Rational zeros Theorem: Let a0 , a1 , · · · an are integers and r be a rational
number satisfying the polynomial equation

an xn + an−1 xn−1 + · · · + a1 x + a0 = 0

where n ≥ 1, an 6= 0, a0 6= 0. Write r = pq where p, q are integers having no


common factors and q 6= 0. Then q divides an and p divides a0 .
Proof:
p p p
an ( )n + an−1 ( )n−1 + · · · + a1 ( ) + a0 = 0
q q q
n n−1 n−1
⇒an p + an−1 p q + · · · + a1 pq + a0 q n = 0

Since p divides an pn + an−1 pn−1 q + · · · + a1 pq n−1 , p will divide a0 q n . This


implies that p divides a0 . The other case follows by similar argument.
• Use Rational zeros Theorem to prove that there is no rational number satis-
fying the equation x2 − 2 = 0. So, length of the diagonal d is not a rational
number.
• Indeed, we need more numbers.

3 Real numbers
Field: Add, substract, Multiply, divide by any nonzero numbers.
Distance: d(a, b) = |b − a| where

a if a ≥ 0,
Absolute value: |a| =
−a if a < 0,

We have

Symmetry: d(a, b) = d(b, a)


Triangle inequality: d(a, b) + d(b, c) ≥ d(a, c)
d(a, b) ≥ 0
d(a, b) = 0 if and only if a = 0.

3
Order property: a ≤ b if b − a is a positive real number.

• Given a and b, either a ≤ b or b ≤ a.

• If a ≤ b and b ≤ a, then a = b.

• Transitivity: if a ≤ b and b ≤ c, then a ≤ c.

• If a ≤ b, then a + c ≤ b + c.

• If a ≤ b and 0 ≤ c then ac ≤ bc.

Interval:

Definition 3.1. A subset X ⊂ R is called an interval if whenever a, b ∈ X and a ≤ c ≤ b,


then c ∈ X.

Any interval will be one of the following type.

[a, b] = {x ∈ R : a ≤ x ≤ b}
(a, b) = {x ∈ R : a < x < b}
[a,b) = {x ∈ R : a ≤ x < b}
(a,b] = {x ∈ R : a < x ≤ b}
( − ∞, b] = {x ∈ R : x ≤ b}
(−∞, b) = {x ∈ R : x < b}
[b, ∞) = {x ∈ R : x ≥ b}
(b, ∞) = {x ∈ R : x > b}

Completeness Axiom:

Definition 3.2. Let S be a nonempty subset of R.


(a) If
S0 ∈ S and s ≤ S0 for all s ∈ S
then we say that S has a largest element S0 . We call S0 the maximum of S and write
S0 = max S.
(b) If
s0 ∈ S and s0 ≤ s for all s ∈ S
then we say that S has a smallest element s0 . We call s0 the minimum of S and write
s0 = min S.

4
Example 3.3. : Let S = [0, 1]. Then S has a largest element 1 and smallest element 0.

Example 3.4. Let S = (0, 1]. Then S has a largest element 1 but S does not have any
smallest element.

Example 3.5. Let S = { n1 : n ∈ N} = {1, 1/2, 1/3, 1/4, · · · , · · · }. Then S has a largest
element 1 but S does not have any smallest element.
1
Example 3.6. Let S = {1 − n
: n ∈ N}. Then S has a smallest element 0 but S does
not have any largest element.

Exercise: Show using induction that if S is a finite subset of R then S has a smallest
element as well as a largest element.

Definition 3.7. Let S be a nonempty subset of R.


(a) If a real number M satisfies s ≤ M for all s ∈ S, then M is called an upper bound
of S and the set S is said to be bounded above.
(b) If a real number m satisfies m ≤ s for all s ∈ S, then m is called an lower bound of
S and the set S is said to be bounded below.
(c) The set S is said to be bounded if it is bounded above and bounded below. Thus S
is bounded if there exist real numbers m and M such that S ⊂ [m, M ].

Definition 3.8. Let S be a nonempty subset of R. Assume that S is bounded above.


We call M the least upper bound of S if

1. s ≤ M for all s ∈ S,

2. whenever M 0 < M , there exists s1 ∈ S such that M < s1 .

We call M the supremum of S and denote it by sup S.

Definition 3.9. Let S be a nonempty subset of R. Assume that S is bounded below.


We call m the greatest lower bound of S if

1. m ≤ s for all s ∈ S,

2. whenever m < m0 , there exists s1 ∈ S such that s1 < m0 .

We call m the infimum of S and denote it by inf S.

Example 3.10. S = (0, 1). Then sup S = 1 and inf S = 0.

5
Remark 3.11. 1. Note that, unlike max S and min S, sup S and inf S need not
belong to S.

2. If S is not bounded above, we define sup S = ∞. If S is not bounded below, we


define inf S = −∞.

3. If X ⊂ Y ⊂ R then
inf Y ≤ inf X ≤ sup X ≤ sup Y.

Completeness Axiom: Every nonempty subset S of R that is bounded above has a


least upper bound. In other words, sup S exists and is a real number.

Remark 3.12. The set of rationals Q does not have this property.

Corollary 3.13. Every nonempty subset S of R that is bounded below has a greatest
lower bound. In other words, inf S exists and is a real number.

Proof : Consider the set −S = {−s : s ∈ S}. Then


(i) −S is bounded above.
(ii) By Completeness Axiom, −S has a least upper bound, say M .
(iii) It is not difficult to see that −M is the greatest lower bound of S (Do it!!). 

Theorem 3.14. Archimedean Property: If a > 0 and b > 0, then for some positive
integer n, we have na > b.

Proof : Assume the Archimedean property fails. Then there exist a > 0 and b > 0 such
that na ≤ b. Consider the set
S = {na : n ∈ N}
Clearly b is an upper bound of S. Let s0 = sup S. Since s0 − a < s0 , there exists n0 ∈ N
such that s0 − a < n0 a. Hence
s0 < (n0 + 1)a.
Since (n0 + 1)a ∈ S, we get a contradiction. 

Remark 3.15. Tale a = 1. The theorem says that given any real number b there exists
a natural number n such that n > b.

Theorem 3.16. Denseness of Q: If a, b ∈ R and a < b, then there is a rational r


such that a < r < b.

Lemma 3.17. If b − a > 1 then there exists an integer m such that a < m < b.

6
Proof : Define r = max{|a|, |b|}. By Archimedean property, there exists an integer K
such that K × 1r > 1 and hence K > r. Now we have,
−K < a < b < K.
Then the set S = {j ∈ Z : −K < j ≤ K and a < j} is finite and nonempty. Let
m = min S.
Then a < m but m − 1 ≤ a. Also, we have
m = m − 1 + 1 ≤ a + 1 < a + (b − a) = b

Proof : ( of theorem: ) We need to show that
m
a< < b ⇔ an < m < bn
n
for some integer m and n where n > 0.
Since b − a > 0, by Archimedean property, there exists n ∈ N such that n(b − a) > 1.
Since bn − an > 1, by lemma, there exists an integer m such that an < m < bn. This
proves the claim. 

Definition 3.18. We call a real number irrational if it is not rational. We denote the
set of irrational by I or Qc .
Remark 3.19. Let r ∈ Q and q ∈ I. Then r + q, rs ∈ I.
Exercise: Prove that if a < b then there exists an irrational number q such that a < q < b.

3.1 Operations of function


Definition 3.20. Let f, g : X → R be real valued functions and let c ∈ R. Then we
define
f + g(x) = f (x) + g(x),
f − g(x) = f (x) − g(x),
f g(x) = f (x)g(x),
f 2 (x) = f (x)2 ,
cf (x) = cf (x),
max{f, g}(x) = max{f (x), g(x)},
min{f, g}(x) = min{f (x), g(x)}.

7
Definition 3.21. Composition Let f : X → Y and g : Y → Z be two functions. We
define g ◦ f : X → Z as
g ◦ f (x) = g(f (x)).

Exercise: Suppose that f (x) = x2 +1 and g(x) = x20 . What are the composite functions
g ◦ f and f ◦ g?

8
4 Sequences
Definition 4.1. A (real) sequence is a function s : N → R. It is customary to write s(n)
as sn and the sequence s as (sn )∞ ∞
n=1 or {sn }n=1 or (sn )n∈N or (sn )n≥1 or (s1 , s2 , s3 , · · · · · · )
or (sn ).

Examples: Consider the sequence (sn )∞


n=1 where

1
1. sn = n

2. sn = (−1)n

3. sn = sin n1
sin n
4. sn = n

5. sn = (1 + 1/n)n

6. sn = last digit of 6n

7. s1 = 1, s2 = 1 and sn = sn−1 + sn−2 for n ≥ 3

8. sn = sin(nπ)
3n
9. sn = n!

Definition 4.2. We say that (sn )n∈N converges to a real number L if for any  > 0 there
exists N ∈ N such that
|sn − L| <  for all n > N.
Equivalently,
L −  < sn < L +  for all n > N.
If sn converges to L then we write limn→∞ sn = L, or sn → L. We call L as a (the) limit
of sequence sn .

Remark 4.3. 1. Limit of a sequence does not depend on finitely many elements of
the sequence.

2. We call (L−, L+) the  neighbourhood (nbd) of L. Given any  > 0, the  neigh-
bourhood (nbd) of L contains all but finitely many elements of the sequence.
Note the difference between “all but finitely many“ and ”infinitely many“.

9
3. In fact, any open interval (a, b) containing L contains all but finitely many
elements of the sequence. (prove it!! by showing that any open interval (a, b)
containing L contains an  nbd of L).

4. Converse is also true. i.e.

• If L is a number such that any open interval (a, b) containing L contains all
but finitely many elements of the sequence then limn→∞ sn = L.

Example 4.4. Let sn = n1 . We will show that sn → 0.

Proof : Take  > 0. By Archimedian property, there exists N ∈ N such that N  > 1.
This implies that for all n > N ,
1
n > 1 ⇒ <  ⇒ |sn − 0| < .
n
This proves the claim. 

Theorem 4.5. A convergent sequence has a unique limit.

Proof : Suppose sn converges to L1 and L2 and L1 6= L2 . Choose  = | L1 −L


4
2
|. Then the
intervals (L1 − , L1 + ) and (L2 − , L2 + ) are disjoint i.e

(L1 − , L1 + ) ∩ (L2 − , L2 + ) = φ (4.1)

Since sn → L1 , there exists N1 such that for n > N1 ,

sn ∈ (L1 − , L1 + )

Similarly since sn → L2 , there exists N2 such that for n > N2 ,

sn ∈ (L2 − , L2 + ).

Take n0 = max{N1 , N2 } + 1. Then

sn0 ∈ (L1 − , L1 + ) and sn0 ∈ (L2 − , L2 + ).

This contradicts (4.1). 


Go through the proof again. Key step is to separate L1 and L2 by disjoint open
intervals.
A sequence that does not converge to any real number is said to diverge.
Examples of divergent sequences: Consider the sequence (sn )∞
n=1 where

10
1. sn = n (tends to infinity!!!)

2. sn = (−1)n (Oscillating between two numbers!!!)

3. sn = sin n1 (Oscillating between infinitely many numbers!!!)

Definition: A sequence (sn )n∈N is said to be a bounded sequence if there exists a real
number M such that
|sn | ≤ M for all n ≥ 1.

Theorem 4.6. A convergent sequence (sn )n∈N is bounded.

Proof : Let (sn )∞


n=1 be a sequence such that sn → L. Choose  = 1. Then there exists N
such that for n > N ,
L − 1 < sn < L + 1.
Take M = max{|s1 |, |s2 |, · · · |sN |, |L| + 1}. It is not difficult to see that

|sn | ≤ M ∀n ∈ N.

Theorem 4.7. Let (sn ) be a sequence in [a, b] (i.e. sn ∈ [a, b]) such that

lim sn = L.
n→∞

Then L ∈ [a, b].

Proof : Assume L < a. Choose  = a − L. Then

(L − , L + ) ∩ [a, b] = (L − , a) ∩ [a, b] = ∅

Hence  nbd of L does not contain any element of the sequence. This contradicts the
fact that limn→∞ sn = L. Therefore we have

L ≥ a.

By similar argument, it follows that L ≤ b. This proves the claim.




Remark 4.8. Note that the key step is to separate L with a and b by open intervals.

Theorem 4.9. Let limn→∞ sn = A and limn→∞ tn = B. Then one has

11
1. limn→∞ sn + tn = A + B,

2. limn→∞ sn − tn = A − B,

3. limn→∞ sn tn = AB,
1 1
4. limn→∞ tn
= B
provided B 6= 0.
sn A
5. limn→∞ tn
= B
provided B 6= 0.

Definition: A sequence (sn )n∈N is said to be

• monotonically increasing if sn ≤ sn+1 for n ≥ 1.

• monotonically decreasing if sn ≥ sn+1 for n ≥ 1.

• monotonic if it is either monotonically increasing or monotonically decreasing.

Theorem 4.10. Suppose (sn )n∈N is monotonic. Then (sn )n∈N converges if and only if
it is bounded.

Proof : Assume that (sn )n∈N is monotonically increasing. Let L = sup{sn : n ∈ N}. To
prove the claim, it is enough to show that sn → L. Take  > 0.
Claim: There exists N such that sN ∈ (L − , L + ).
Proof of claim: If not then L −  would have been an upper bound of {sn : n ∈ N}.
For n > N , we have

L −  < sN ≤ sn ≤ L ⇒ sn ∈ (L − , L + ).

The other case will follow similarly. 

Theorem 4.11. Sandwich Theorem: Let (rn )n∈N , (sn )n∈N and (tn )n∈N be sequences
of real numbers. Let

(i) rn ≤ sn ≤ tn for all n ∈ N,


(ii) lim rn = lim tn = L.
n→∞ n→∞

Then
lim sn = L.
n→∞

12
Proof : Take  > 0. Since limn→∞ rn = L there exists N1 such that for n > N1 , we have

L −  < rn < L + .

Since limn→∞ tn = L, there exists N2 such that for n > N2 , we have

L −  < tn < L + .

Take N = max{N1 , N2 }. For n > N , we have

L −  < rn ≤ sn ≤ tn < L + .

This proves that


lim sn = L.
n→∞

Alternative proof: Since  nbd of L contains all but finitely many elements of (rn )
and (tn ) it will contain all but finitely many elements of the sequence (sn ) as well. This
proves that
lim sn = L.


Remark 4.12. If the condition holds for all but finitely many n then also the result
holds true (prove it!!).
cosn −1 cosn −1
Example 4.13. n
→ 0 because n
≤ n
≤ n1 , n
→ 0 and 1
n
→ 0.

Definition 4.14. 1. We say that sequence (sn ) diverges to ∞ if for all M ∈ N there
exists N such that for n > N , we have

sn > M.

We denote it by limn→∞ sn = ∞ or sn → ∞.

2. We say that sequence (sn ) diverges to −∞ if for all M ∈ N there exists N such
that for n > N , we have
sn < −M.
We denote it by limn→∞ sn = −∞ or sn → −∞.

Warning!! Apply the limit theorems carefully. First verify by yourself and then
apply.

13
Example 4.15. Take the sequences rn = n, sn = −n and tn = −n + 1. Then rn → ∞,
sn , tn → −∞ but rn + sn → 0 and rn + tn → 1.

Exercise: Let limn→∞ sn = ∞ and limn→∞ tn = L ∈ R. Then

(i) lim sn + tn = ∞.
n→∞
(ii) lim sn − tn = ∞.
n→∞

Exercise: Suppose (sn )n∈N is a monotonically increasing unbounded sequence. Then

lim sn = ∞.

Exercise: Suppose (sn )n∈N is a monotonically decreasing unbounded sequence. Then

lim sn = −∞.

Remark 4.16. • A monotonic increasing real sequence will either converge to a real
number or diverge to ∞.

• A monotonic decreasing real sequence will either converge to a real number or


diverge to −∞.

Exercise: Let (sn )n∈N and (tn )n∈N be sequences such that sn ≤ tn for all n ∈ N. Let

lim sn = S and lim tn = T

Then
S ≤ T.

5 Subsequence
Definition 5.1. Given a sequence (sn )∞ ∞
n=1 , consider a sequence (nk )k=1 of natural num-
bers such that
n1 < n2 < n3 < · · · · · · .
Then the sequence (snk )∞ ∞
k=1 is called a subsequence of the sequence (sn )n=1 .

Example 5.2. If (s1 , s2 , s3 , · · · ) is a sequence then (s1 , s3 , s5 · · · ) and (s6 , s16 , s26 , · · · )
are subsequences of (sn ).

14
Example 5.3. Let sn = (−1)n . Then (s2n ) = (1, 1, 1, 1, · · · ) is a subsequence of (sn ).

Definition 5.4. Let (sn ) be a sequence in R. A subsequential limit or limit point of (sn )
is any real number or symbol ∞ or −∞ that is the limit of some subsequence of (sn ).

Example 5.5. Take the sequence sn = (−1)n . Since the subsequences limk→∞ s2k = 1
and limk→∞ s2k+1 = −1, 1 and −1 are subsequential limit (limit point) of (sn ).
Example 5.6. For the sequence sn = (−1)n n, ∞ and −∞ are limit points of (sn ).
Theorem 5.7. If a sequence (sn ) converges to L, then every subsequence converges to
the the same limit L. (i.e. a convergent sequence has a unique limit point).
Proof : Take a subsequence (snk ) and let  > 0. Since (sn ) converges to L, there exists
N such that for n > N , we have
|sn − L| < .
Note that nk ≥ k for all k. Hence for k > N , we have nk > N and hence
|snk − L| < .
This proves the claim.

Alternative proof: Since  nbd of L contains all but finitely many elements of
(sn ), it will contain all but finitely many elements of the subsequence (snk ) as well. This
proves that
lim snk = L.


Remark 5.8. The results holds true if lim sn = ∞ or lim sn = −∞.

5.1 limsup and liminf


Let (sn )n∈N be a sequence. Define
A1 = sup{s1 , s2 , s3 , · · · · · · }
A2 = sup{s2 , s3 , s4 · · · · · · }
A3 = sup{s3 , s4 , s5 · · · · · · }
·····················
·····················
An = sup{sn , sn+1 , sn+2 , · · · · · · }

15
Clearly A1 ≥ A2 ≥ A3 ≥ · · · · · · i.e. (An )n∈N is a monotonically decreasing sequence.
Define 
∞ if An = ∞ ∀n,
lim inf sn =
n→∞ limn→∞ An otherwise .

Remark 5.9. • Note that limn→∞ An can be a real number or symbol ∞ or symbol
−∞.

• limn→∞ An = ∞ if An = ∞ for all n ∈ N. e.g. sn = n.

• limn→∞ An ∈ R if An ’s are bounded below. e.g. sn = 1 + n1 .

• limn→∞ An = −∞ if An ’s are not bounded below. e.g. sn = −n.

Similarly, define

a1 = inf{s1 , s2 , s3 , · · · · · · }
a2 = inf{s2 , s3 , s4 · · · · · · }
a3 = inf{s3 , s4 , s5 · · · · · · }
·····················
·····················
an = inf{sn , sn+1 , sn+2 , · · · · · · }

Clearly (an )n∈N is a monotonically increasing sequence. Define



−∞ if an = −∞ ∀n,
lim inf sn =
n→∞ limn→∞ an otherwise .

Remark 5.10. • Note that limn→∞ an can be a real number or symbol ∞ or symbol
−∞.

• limn→∞ an = −∞ if an = −∞ for all n ∈ N. e.g. sn = −n.

• limn→∞ an ∈ R if an ’s are bounded above. e.g. sn = 1 − n1 .

• limn→∞ an = ∞ if an ’s are not bounded above. e.g. sn = n.

Example 5.11. 1. Take the sequence sn = (−1)n . In this case An = 1 and an = −1.
Hence lim sup sn = 1 and lim inf sn = −1.

2. Let sn = n1 . Then An = 1
n
and an = 0. So, lim sup sn = lim inf sn = 0.

16
Exercise: Let (sn ) be a sequence in [a, b]. Show that

lim sup sn , lim inf sn ∈ [a, b].


n→∞ n→∞

Exercise: Let (sn ) be a sequence in [a, ∞). Show that

lim sup sn , lim inf sn ∈ [a, ∞) ∪ {∞}.


n→∞ n→∞

Exercise: Let (sn ) be a sequence in (−∞, a]. Show that

lim sup sn , lim inf sn ∈ (−∞, a] ∪ {−∞}.


n→∞ n→∞

Exercise: Prove that if M > lim sup sn then there exists N such that for n > N ,

sn < M.

Exercise: Prove that if m < lim inf sn then there exists N such that for n > N ,

sn > m.

Exercise: If lim sup < ∞ and lim inf < ∞ then the sequence (sn ) is bounded.

Theorem 5.12. Let (sn ) be a real valued sequence.


(i) If lim sn = L ∈ R then

lim sup sn = lim inf sn = lim sn = L

(ii) If lim sup sn = lim inf sn = L ∈ R then

lim sn = L

Remark 5.13. The statement holds true for L = ∞ or −∞.

Proof : Define

An = sup{sn , sn+1 , · · · · · · } and an = inf{sn , sn+1 , · · · · · · }.

So, lim An = lim sup sn and lim an = lim inf sn .


Proof: (⇒) Take  > 0. There exists N such that for n > N ,

sn < L + .

17
Hence for all n > N ,
An ≤ L + .
Taking limit as n → ∞, we get

lim sup sn ≤ L + .

Since  is arbitrary, we get


lim sup sn ≤ L.
A similar argument shows that
lim inf sn ≥ L.
Since lim sup sn ≥ lim inf sn , we have

lim sup sn = lim inf sn = L.

Proof: (⇐) Let  > 0. There exists N0 such that for n > N0 , we have

|L − An | < 

Hence
sup{sn : n > N0 } < L +  ⇒ sn < L +  for n > N0 .
Similarly since lim an = L, there exists N1 such that for n > N1 , we have

inf{sn : n > N0 } > L −  ⇒ sn > L −  for n > N1 .

Take N = max{N0 , N1 }. For n > N , we have

L −  < sn < L + .

This proves the claim. 

Proposition 5.14. For a sequence (sn )n∈N , let

A = lim sup sn and B = lim inf sn .


n→∞ n→∞

Then there exist subsequences (snj )j∈N and (snk )k∈N such that

lim snj = A, lim snk = B


j→∞ k→∞

18
Proof : Assume that sn1 , sn2 , sn3 , · · · snj−1 , have been choosen. Since

anj = sup{snj−1 +1 , snj−1 +2 , · · · },

we can choose snj ∈ (snj−1 +1 , snj−1 +2 , · · · ) such that

1 1
snj ∈ (anj − , anj + ).
j j

So, we have a subsequence (snj )j∈N of (sn )n∈N .


claim: we have
lim snj = A
n→∞

Proof : Fix  > 0. There exists K1 such that for k > K1 ,



|A − ak | ≤ .
2
1
Choose K2 such that K2
< 2 . Take K = max{K1 , K2 }. Then for k > K, we have
 
|A − snk | ≤ |A − ank | + |ank − snk | < + =
2 2


Theorem 5.15. Let (sn ) be any sequence in R, and let S denote the set of subsequential
limits of (sn ). Then one has,
(i) S is nonempty.
(ii) sup S = lim sup(sn ) and inf S = lim inf(sn ).
(iii) limn→∞ sn exists if and only if S has exactly one element, namely limn→∞ sn .

Remark 5.16. Here limn→∞ sn can be equal to ∞ or −∞.

Proof : (i) follows from previous theorem. To prove (ii), consider any subsequential limit
t of a subsequence (snk ) of (sn ). We have

t = lim inf snk = lim sup snk .

Since
{snk : k > N } ⊂ {sn : n > N }
for each n ∈ N, we have

lim inf sn ≤ lim inf snk = t = lim sup snk ≤ lim sup sn .

19
This inequality holds for all t in S; therefore

lim inf sn ≤ inf S ≤ sup S ≤ lim sup sn .

Proposition 5.14 shows that lim inf sn , lim sup sn ∈ S. Therefore (ii) holds. (iii) follows
from Theorem 5.12. 

Theorem 5.17. (Bolzano-Weierstrass Theorem:) Every bounded sequence has a


convergent subsequence.

Proof : Let (sn )n∈N be a bounded sequence and let A = lim supn→∞ sn . Since (sn )n∈N
is bounded, we get A < ∞. By Proposition 5.14, we get a subsequence of (sn )n∈N
converging to A. 

6 Cauchy sequence
Definition 6.1. A sequence (sn )∞
n=1 is called a Cauchy sequence if for every  > 0 there
exists N such that
|sn − sm | <  whenever n, m ≥ N.

Theorem 6.2. A Cauchy sequence is a bounded sequence.

Proof : Let (sn )n∈N be a Cauchy sequence. Choose  = 1. Then there exists N such that
for m > N ,
|sN − sm | < 1.
Take M = max{|s1 |, |s2 |, · · · |sN |, |sN | + 1}. Hence

|sn | ≤ M ∀n ∈ N.

Theorem 6.3. A Cauchy sequence having a convergent subsequence, converges.

Proof : Let (sn )n∈N be a Cauchy sequence and (snk )k∈N be its subsequence such that

snk → l as k → ∞.

We will show that sn → l. Fix  > 0. There exists N0 such that for n, m > N0 ,

|sn − sm | < .
2

20
Since (snk ) → l, there exists M such that for k > M ,

|snk − l| < .
2
Let N = max{N0 , nM +1 }. Then for n > N , we have
 
|sn − l| ≤ |sn − snM +1 | − |snM +1 − l| < + = .
2 2


Theorem 6.4. A convergent sequence is a Cauchy sequence.

Proof : Let (sn )n∈N be a sequence conveging to s. Take  > 0. Since sn → s, there exists
N such that for n > N , we have

|sn − s| < .
2
Hence for n, m > N , we have

|sn − sm | < |sn − s| + |s − sm | < .


What about the converse!! For a real sequence, converse is true.

Theorem 6.5. (Cauchy criterion:) Every Cauchy sequence converges.

Proof : Let (sn )n∈N be a Cauchy sequence.


Step I: By Theorem 6.2, the sequence (sn )n∈N is bounded.
Step II: By Bolzano Weierstrass Theorem, the sequence (sn )n∈N has a convergent sub-
sequence.
Step III: By Theorem 6.3, the sequence (sn )n∈N converges. 

7 Series
Aim: want to make sense of infinte sum!!!!
Given a sequence (an )n∈N , we use the following notation.
q
X
an = ap + ap+1 + ap+2 + · · · + aq .
n=p

The symbol

X X
an or a1 + a2 + a3 + · · · or an
n=1

21
is called an infinte series or just a series. With (an )n∈N , we associate a sequence (sn )n∈N
where n
X
sn = am = a1 + a2 + · · · + an .
m=1
P∞
These are called partial sums of the series n=1 an . If (sn )n∈N converges to s then we
say that the series converges and write

X
an = s.
n=1
P∞
Similarly we can make sense of n=p an . If (sn )n∈N diverges then we say that the series
P∞
n=1 an diverges.

Example 7.1. Consider the geometric series;



X
S= an .
n=0

The nth partial sum sn is given by


1 − an+1
sn = .
1−a
1
If |a| < 1 then sn → 1−a
If |a| ≥ 1 then the series diverges.
.

Theorem 7.2. (Cauchy criterion): The series ∞


P
n=1 an converges if and only if for
every  > 0, there exists N ∈ N such that
n
X
| ak | < 
k=m

if n ≥ m ≥ N .

Proof : By a direct application of Theorem 6.5 (Cauchy criterion) and the fact that
|sn − sm−1 | = | nk=m ak |, the claim follows.
P


P∞
Theorem 7.3. If series n=1 an converges then an → 0.

Proof : Use previous theorem. 


Qn: Does the converse hold?
Ans: No!! Take the series ∞ 1 1
P
n=1 n . Here n
→ 0 but the series diverges (we will prove it
later).

22
Theorem 7.4. A series of nonnegative terms converges if and only if the sequence of
partial sums is bounded.

Proof : Sequence of partial sums of a series of nonnegative terms is monotonically in-


creasing. Hence the result. 
P
Theorem 7.5. Suppose a1 ≥ a2 ≥ a3 ≥ · · · · · · ≥ 0. Then the series an converges if
and only if the series

X
2k a2k = a1 + 2a2 + 4a4 + 8a8
k=0
converges.

Proof : It suffices to proved boundedness of partial sums. Let

s n = a1 + a2 + a3 · · · + an
tk = a1 + 2a2 + 4a4 + · · · + 2k a2k .

For n > 2k ,

sn ≥ a1 + a2 + (a3 + a4 ) + (a5 + a6 + a7 + a8 ) + · · · (a2k−1 +1 + · · · + a2k )


1
≥ a1 + a2 + 2a4 + · · · + 2k−1 a2k
2
1
= tk
2
On the other hand, if n < 2k ,

sn ≤ a1 + (a2 + a3 ) + (a4 + a5 + a6 + a7 ) + · · · (a2k + a2k +1 + · · · + a2k+1 −1 )


≤ a1 + 2a2 + 4a4 + · · · + 2k a2k
= tk

Therefore, the sequences (sn ) and (tk ) are either both bounded or both unbounded. This
proves the claim.


Theorem 7.6. The series


X 1
np
converges if p > 1 and diverges if p ≤ 1.

Proof : Apply previous theorem. 

23
Theorem 7.7. Comparison test:
(i) If |an | ≤ cn for n > N0 , and if ∞
P P∞
n=1 cn converges, then n=1 an converges.
(ii) If an ≥ dn ≥ 0 for n > N0 , and if n=1 dn diverges, then ∞
P∞ P
n=1 an diverges.
P∞ P∞
Corollary 7.8. If n=1 |an | converges, then n=1 an converges.

Proof : Given  > 0, there exists N such that for n ≥ m ≥ N , we have


n
X
ck < 
k=m

by the Cauchy criterion. Take M = max{N, N0 }. Then for n ≥ m ≥ N , we have


Xn X n n
X
ak ≤ |ak | ≤ ck ≤ .


k=m k=m k=m
P∞ P∞
Hence (i) follows. The other claim follows from (i), for if n=1 an converges then n=1 dn
converges. 
P∞ P∞
Definition 7.9. If n=1 |an | converges, we say that the series n=1 an converges abso-
lutely.
P∞ n1
Example 7.10. n=1 (−1) n converges but does not converge absolutely.
P
Theorem 7.11. Ratio test: The series an
an+1
(i)converges absolutely if lim supn→∞ an < 1,

(ii)diverges if lim inf n→∞ an+1 > 1,

an

Remark 7.12. In the remaining case, i.e. lim inf n→∞ an+1
an+1
≤ 1 ≤ lim sup ,

an n→∞ an
the test gives no information.

Example 7.13. Consider the geometric series


X
an = 1 + a + a2 + · · ·

Here
an+1
lim sup | | = |a|.
an
So, by Ratio test
(i) if |a| < 1, the series converges;
(ii) if |a| > 1, the series diverges.

24
Example 7.14. Consider the series
X n
.
n2 +3
Here
an+1 n+1 n2 + 3 n + 1 n2 + 3
= = .
an (n + 1)2 + 3 n n n2 + 2n + 4
Hence lim | an+1
an
| = 1. So the test gives no information. Now

n n 1
≥ = .
n2 + 3 n2 + 3n2 4n
1
P P n
Since n
diverges, by Comparison test, the series n2 +3
diverges.

Example 7.15. Consider the series


X 1
.
n2 +1
Here
an+1 n2 + 1
lim = = 1.
an (n + 1)2 + 1
So the test gives no information. Now
1 1
≤ .
n2 + 1 n2
1 1
P P
Since n2
converges, by Comparison test, the series n2 +1
converges.

Proof : 
P 1
Theorem 7.16. Root test: For the series, an , let α = lim supn→∞ |an | n . Then the
P
series an
(i)converge abslolutely if α < 1,
(ii)diverges if α > 1.

Remark 7.17. For α = 1, the test gives no information.

Proof : 

Example 7.18. Consider the series


X (−1)n
√ .
n

25
Both Ratio test and Root test do not give any information as
1 an+1
lim sup |an | n = lim sup | | = 1.
an
P 1
Since √ diverges, we will not be able to use comparision test. It turns out that this
n
series converges by the Alternating Series test.
Theorem 7.19. Alternating series Theorem:
If a1 ≥ a2 ≥ a3 ≥ · · · · · · ≥ 0 and lim an = 0 then the alternating series
X
(−1)n an = a1 − a2 + a3 − a4 + · · ·
converges.
Proof : We will show that for n ≥ m > N ,
Xn
(−1)k ak ≤ aN . (7.1)


m

Assuming this, the claim will follow by Cauchy criterion.


To prove (7.1), we fix n ≥ m > N and define
A = am − am+1 + am+2 − am+3 + · · · ± an
so that n
X
(−1)k ak = (−1)m A.
m
If n − m is odd, the last term of A is −an , so
A = (am − am+1 ) + (am+2 − am+3 ) + · · · + (an−1 − an ) ≥ 0
and also
A = am − (am+1 − am+2 ) − (am+3 − am+4 ) − · · · − (an−2 − an−1 ) − an ≤ am
If n − m is even, the last term of A is an , so
A = (am − am+1 ) + (am+2 − am+3 ) + · · · + (an−2 − an−1 ) + an ≥ 0
and also
A = am − (am+1 − am+2 ) − (am+3 − am+4 ) − · · · − (an−1 − an ) ≤ am .
In either case, we have 0 ≤ A ≤ am . Hence
Xn
(−1)k ak = A ≤ am ≤ aN .


m

26
7.1 Power series
Definition 7.20. Given any sequence of real numbers (an )∞
n=0 , the series


X
an x n = a0 + a1 x + a2 x 2 + · · ·
n=0

is called power series centred at 0. More generally, the series



X
an (x − c)n = a0 + a1 (x − c) + a2 (x − c)2 + · · ·
n=0

is called power series centred at c.

Remark 7.21. Note that any polynomial is a power series.


P∞
Theorem 7.22. For the power series n=0 an xn let

1 1
β = lim sup (|an |) n and R= .
n→∞ β

[If β = 0 we set R = ∞, and if, β = ∞ then we set R = 0.] Then


(i) the power series converges for |x| < R;
(ii) the power series diverges for |x| > R.

Proof : Apply root test. 

Remark 7.23. • If lim |a|an+1


n|
|
exists, then this limit equals β.

• R is called radius of convergence.

• At x = R or −R, the series may converge or diverge.

Example 7.24. Consider



X
xn .
n=0

Here
β=1 and R = 1.
So for |x| < 1, the series converges and for |x| > 1, the series diverge. For x = 1 or −1,
the series diverges.

27
Example 7.25. Consider

X 1 n
x .
n=0
n
Since
|an+1 | n
lim = lim = 1,
|an | n+1
we have
β=1 and R = 1.
So for |x| < 1, the series converges and for |x| > 1, the series diverge. For x = 1, the
series diverges and for x = −1, the series converges by Alternating Series Test.

Example 7.26. Consider



X 1 n
x .
n=0
n!
Since
|an+1 | n! 1
lim = lim = lim = 0,
|an | n + 1! n+1
we have
β=0 and R = ∞.
Hence the series converges for all x ∈ R.

8 Continuity
Throughout this course, we will be discussing real valued functions whose domain is a
subset of R e.g. (0, 1), [0, 1], (0, 1], Q etc. We generally denote by X the domain of a
function.

Definition 8.1. Let X ⊂ R and p ∈ R. We call p a limit point of X if there exists a


sequence (sn ) such that

sn ∈ X ∀n, sn 6= p and sn → p.

Exercise: Show that p a limit point of X if and only if every open interval containing
p intersects X at some point other than p.
Exercise: Show that 1 is a limit point of (0, 1).
Exercise: Let p ∈ (a, b). Show that p is a limit point of (a, b).
Exercise: Show that 1 is not a limit point of N.

28
Definition 8.2. Let f : X → R be a function and let p is a limit point of X. We write

lim f (x) = q or f (x) → q as x → p


x→p

if for  > 0 there exists δ > 0 such that

|f (x) − q| <  whenever x ∈ X and 0 < |x − p| < δ.

Remark 8.3. The value f (p) does not play any role here. In fact, f may not be defined
at p.

Theorem 8.4. Let X, f , p be as in Definition 8.2. Then

lim f (x) = q
x→p

if and only if
lim f (pn ) = q.
n→∞

for every sequence (pn ) in X such that

pn 6= p and lim pn = p.
n→∞

Corollary 8.5. If f has a limit at p, then it is unique.

Theorem 8.6. Let limx→p f (x) = A and limx→p g(x) = B. Then one has

1. limx→p (f + g)(x) = A + B,

2. limx→p (f − g)(x) = A − B,

3. limx→p (f g)(x) = AB,

4. limx→p fg (x) = A
B
, provided B 6= 0.

Proof : Follows from Theorem 8.4. 

Definition 8.7. Let f : X → R be a function and let x0 ∈ X. We say that f is


continuous at x0 if if for  > 0 there exists δ > 0 such that

|f (x) − f (x0 )| <  whenever x ∈ X and |x − x0 | < δ.

A function is said to be continuous if it is continuous at each point of its domain.

29
Remark 8.8. Here f must be defined at x0 . If x0 is a limit point of X then f is
continuous at x0 if and only if

lim f (x) = f (x0 ).


x→x0

If x0 is not a limit point of X then f is always continuous at x0 . e.g. If f : N → R then


f is always continuous at each point of N.

Theorem 8.9. Let X, f , p be as above. Then f is continuous at x0 if and only if

lim f (xn ) = f (x0 ).


n→∞

for every sequence (xn ) in X such that

lim xn = x0 .
n→∞

Exercise: Prove the above remark using this theorem.

Theorem 8.10. If f and g be two functions that are continuous at x0 then

• f + g, f − g and f g are continuous at x0 ,


f
• g
is continuous at x0 if g(x0 ) 6= 0.

Proof : Use Sequential definition. 

Theorem 8.11. If f is continuous at x0 and g is continuous at f (x0 ) then g ◦ f is


continuous at x0 .

Proof : Use Sequential definition. 

Theorem 8.12. Let f be a continuous real-valued function on a closed interval [a, b].
Then f is a bounded function. Moreover, f attains its maximum and minimum values
on [a, b]; that is, there exist x0 , y0 ∈ [a, b] such that f (x0 ) ≤ f (x) ≤ f (y0 ) for all x ∈ [a, b]
.

Proof : Assume that f is not bounded above. Then to each n ∈ N there corresponds an
xn ∈ [a, b] such that
f (xn ) > n.

30
By the Bolzano-Weierstrass theorem, (xn ) has a subsequence (xnk ) that converges to
some real number x0 . The number x0 also must belong to the closed interval [a, b]. Since
f is continuous at x0 , we have

lim f (xnk ) = f (x0 )


k→∞

but we also have


lim f (xnk ) = ∞,
k→∞

which is a contradiction. It follows that f is bounded above. Apply the same argument
to −f to show that f is bounded below. Hence f is a bounded function.
Let M = sup{f (x) : x ∈ [a, b]}. As we showed M < ∞. For each n ∈ N there exists
yn ∈ [a, b] such that
1
M − < f (yn ) ≤ M.
n
By Bolzano-Weierstrass theorem, (yn ) has a subsequence (ynk ) that converges to a point
y0 ∈ [a, b]. We have
f (ynk ) → M
Since f is continuous, we get

f (y0 ) = lim f (xnk ) = M.


k→∞

This proves that f attains its maximum. To prove the remaining part, use same argument
to −f .


Theorem 8.13. Intermediate Value Theorem. If f is a continuous real-valued


function on an interval I, then f has the intennediate value property on I: Whenever
a, b ∈ I, a < b and y lies between f (a) and f (b) i.e., either f (a) < y < f (b) or f (b) <
y < f (a), there exists at least one x ∈ (a, b) such that f (x) = y.

Proof : Assume that f (a) < y < f (b); the other case is similar. Let

S = {x ∈ [a, b] : f (x) < y}.

Note that S is nonempty as a ∈ S. Define x0 = sup S ∈ [a, b]( why?).


claim: There exists a sequence (sn ) in S converging to x0 .
Proof : exercise 
claim: f (x0 ) ≤ y.

31
Proof : Since sn ∈ S, f (sn ) < y. Taking limit, we get the claim. 
For each n ∈ N, define tn = min{x0 + n1 , b}. Since tn ∈
/ S, we have f (tn ) ≥ y. Moreover,
since
1
x0 ≤ tn ≤ x0 +
n
we have
lim tn = x0 .
n→∞

As f is continuous, we get
f (x0 ) = lim f (tn ) ≥ y.
n→∞

This proves the claim.




Definition 8.14. Let f be a real-valued function defined on a set S ⊂ R. Then f is


uniformly continuous on S if for each  > 0 there exists δ > 0 such that x, y ∈ S and
|x − y| < δ imply |f (x) − f (y)| < . We will say that f is uniformly continuous if f is
uniformly continuous on dom(f ).

Remark 8.15. 1. Note that δ depends only on , not on x and y.

2. If a function is uniformly continuous on its domain, then it must be continuous on


its domain.

Exercise: If f is uniformly continuous on a set S and (sn ) is a Cauchy sequence in S,


then (f (sn )) is a Cauchy sequence.

Theorem 8.16. If f is continuous on a closed interval [a, b], then f is uniformly con-
tinuous on [a, b].

Proof : Assume that f is not uniformly continuous on [a, b]. Then for each δ > 0 there
exist x, y ∈ [a, b] such that |x − y| < δ and |f (x) − f (y)| ≥ . Then for each n ∈ N
there exist xn , yn in [a, b] such that |x − y| < n1 and |f (x) − f (y)| ≥ . By the Bolzano-
Weierstrass theorem, a subsequence (xnk ) of (xn ) converges. Moreover, if x0 = lim xnk ,
then x0 ∈ [a, b].
Claim: lim ynk = x0 . Proof : Given ε > 0, there exists K1 such that for k > K1 , we
have
ε
|xnk − x0 | <
2

32
1
Choose K2 ∈ N such that K2
< 2ε . Define K = max{K1 , K2 }. Then for k > K, we have

1 ε ε ε
|ynk − x0 | ≤ |ynk − xnk | + |xnk − x0 | < + < + < ε.
k 2 2 2

Since f is continuous at x0 , we have

f (x0 ) = lim f (xnk ) = lim f (ynk ).

Hence
lim f (xnk − f (ynk ) = 0.
Since |f (xnk − f (ynk )| ≥  for all k, we get a contradiction. 

33
9 Differentiation
Let f : (a, b) → R be a function and let c ∈ (a, b). We say that f is differentiable at c,
or that f has a derivative at p, if the limit
f (x) − f (c)
lim
x→c x−c
exists and is finite. We will write f 0 (c) for the derivative of f at c.

f (x) − f (c)
f 0 (c) = lim
x→c x−c
whenever this limit exists and is finite.

Remark 9.1. Differentiation is a local property i.e. if f = g in any open interval


(arbitrary small!!) containing a then f 0 (a) = g 0 (a).

Example: Let f : R → R be function given by x 7→ xn where n is a natural number.


Then
x n − cn (x − c)(xn−1 + xn−2 c + xn−3 c2 + · · · + xcn−2 + cn−1 )
lim = lim
x→c x − c x→c x−c
= lim xn−1 + cxn−2 + c2 xn−3 + · · · + cn−2 x + cn−1
x→c
n−1
= nc .

Ecercise: Let f : (−1, 1) → R be function given by



1 if x ∈ Q ∩ (−1, 1),
f (x) =
0 otherwise .

Show that f is not differentiable at 0.


Ecercise: Let f : (−1, 1) → R be function given by

x if x ∈ Q ∩ (−1, 1),
f (x) =
0 otherwise .

Show that f is differentiable at 0 and find f 0 (0).


Ecercise: Let f : (−1, 1) → R be function given by

x2 sin 1 if x 6= 0,
x
f (x) =
0 if x = 0.

34
Show that f is differentiable at 0 and find f 0 (0).
Ecercise: Let f : (−1, 1) → R be function given by

x sin 1 if x 6= 0,
x
f (x) =
0 if x = 0.

Show that f is continuous at 0 but f is not differentiable at 0.

Theorem 9.2. If f is differentiable at a point c, then f is continuous at c.

Proof : Need to show that limx→c f (x) = f (c). We have


 f (x) − f (c) 
lim f (x) = lim (x − c) + f (c)
x→c x→c x−c
f (x) − f (c)
= lim(x − c) lim + lim f (c)
x→c x→c x−c x→c
0
= 0 × f (c) + f (c) = f (c)

Theorem 9.3. Let f and g be functions that are differentiable at the point a. Each of
the functions cf , (c ∈ R), f + g and f g is also differentiable at a. If g(a) 6= 0 then f /g
is differentiable at a. The formulas are
(i) (cf )0 (a) = cf 0 (a);
(ii) (f + g)0 (a) = f 0 (a) + g 0 (a);
(iii) (f g)0 (a) = f (a)g 0 (a) + f 0 (a)g(a); (product rule)
g(a)f 0 (a)−f (a)g 0 (a)
(iv) (f /g)0 (a) = g 2 (a)
if g(a) 6= 0 (quotient rule).

Theorem 9.4. [Chain rule]: If f is differentiable at a and g is differentiable at f (a),


then the composite function g ◦ f is is differentiable at a and (g ◦ f )0 (a) = g 0 (f (a))f 0 (a).

Proof : 

Theorem 9.5. If f is defined on an open interval containing x0 and if f assumes its


maximum or minimum at x0 and if f is differentiable at x0 then f 0 (x0 ) = 0.

Proof : Assume f is defined on (a, b) and x0 ∈ (a, b). Suppose f attains its maximum at
x0 .
Step I: Choose a sequence (sn ) in (a, b) such that

s n ≥ x0 and sn → x0 .

35
(one possible choice is; sn = min{x0 + n1 , b})
We have
f (x) − f (x0 ) f (sn ) − f (x0 )
f 0 (x0 ) = lim = lim ≤ 0.
x→x0 x − x0 n→∞ s n − x0
Step II: Choose a sequence (tn ) in (a, b) such that

tn ≤ x0 and tn → x0 .

(one possible choice is; tn = max{x0 − n1 , a})


We have
f (x) − f (x0 ) f (tn ) − f (x0 )
f 0 (x0 ) = lim = lim ≥ 0.
x→x0 x − x0 n→∞ tn − x0
Therefore we have, f 0 (x0 ) = 0. The other case can be proved using similar argument or
replacing f by −f . 

Theorem 9.6. Rolle’s Theorem: Let f be a continuous function on [a, b] that is


differentiable on (a, b) and satisfies f (a) = f (b). There exists c in (a, b) such that f 0 (c) =
0.

Proof : Since f is continuous on [a, b], there exists x0 and y0 in [a, b] such that

f (x0 ) ≤ f (x) ≤ f (y0 ) for x ∈ [a, b].

If x0 and y0 are endpoints of [a, b] then f is a constant function and hence f 0 (x) = 0 for
all x ∈ (a, b). Otherwise, f assumes either a maximum or a minimum at c in (a, b) and
by previous theorem f 0 (c) = 0. 

Theorem 9.7. Mean value Theorem: Let f be a continuous function on [a, b] that
is differentiable on (a, b). There exists c in (a, b) such that

f (b) − f (a)
f 0 (c) = .
b−a
(Slanted version of Rolle’s Theorem).

Proof : Define L : [a, b] → R given by

f (b) − f (a)
L(x) = (x − a) + f (a) − f (x).
b−a
Then
L(a) = L(b) = 0

36
and
f (b) − f (a)
L0 (x) = − f 0 (x)
b−a
for x ∈ (a, b). Applying Rolle’s Theorem, there exists c ∈ (a, b) such that

f 0 (c) = 0.

Corollary 9.8. Let f be a differentiable function on (a, b) such that f 0 (x) = 0 for
x ∈ (a, b). Then f is a constant function on (a, b).

Proof : If f is not a constant function on (a, b) then there exists x1 < x2 in (a, b) such
that f (x1 ) 6= f (x2 . By the Mean Value Theorem, there exists c ∈ (x1 , x2 ) such that
f (x2 ) − f (x1 )
f 0 (c) = 6= 0,
x2 − x1
and we get a contradiction. 

Corollary 9.9. Let f and g be differentiable functions on (a, b) such that f 0 = g 0 on


(a, b). Then there exists a constant c such that f (x) = g(x) + c for all x ∈ (a, b).

Proof : Apply previous corollary to the function f − g. 

Definition 9.10. Let f be a real-valued function defined on an interval I. We say that


f is strictly increasing on I if

x1 , x2 ∈ I, x1 < x2 ⇒ f (x1 ) < f (x2 ),

strictly decreasing on I if

x1 , x2 ∈ I, x1 < x2 ⇒ f (x1 ) > f (x2 ),

increasing on I if
x1 , x2 ∈ I, x1 < x2 ⇒ f (x1 ) ≤ f (x2 ),
decreasing on I if
x1 , x2 ∈ I, x1 < x2 ⇒ f (x1 ) ≥ f (x2 ).

Corollary 9.11. Let f be a differentiable function on an interval (a, b). Then


(i) f is strictly increasing if f 0 (x) > 0 for all x ∈ (a, b);
(ii) f is strictly decreasing if f 0 (x) < 0 for all x ∈ (a, b);
(iii) f is increasing if f 0 (x) ≥ 0 for all x ∈ (a, b);
(iv) f is decreasing if f 0 (x) ≤ 0 for all x ∈ (a, b);

37
Proof : Take x1 , x2 ∈ (a, b) such that x1 < x2 . By Mean value Theorem, for some
c ∈ (a, b), we have
f (x2 ) − f (x1 )
= f 0 (c) > 0.
x2 − x1
Hence
f (x2 ) < f (x1 ).
The remaining cases follow by similar argument. (do it!!). 

9.1 Taylor Series


Let f be a function defined on some open interval containing a. If f possesses derivatives
of all orders at 0, then the series

X f (k) (a)
(x − a)k
k=0
k!

is called the Taylor series for f about a. The remainder Rn (x, a) is defined by
n−1 (k)
X f (a)
Rn (x) = f (x) − (x − a)k .
k=0
k!

For any x,

X f (k) (a)
f (x) = (x − a)k if and ony if lim Rn (x, a) = 0.
k=0
k! n→∞

Here we will only discuss Taylor series around 0. We will denote the remainder Rn (x, 0)
by Rn (x). Example: Define f : R → R given by

e− x1 if x > 0,
f (x) =
0 if x ≤ 0,

Show that f (n) (x) = 0 for all n ∈ N. So, f does not agree with its Taylor series around
0; in other words,

X f (k) (a)
f (x) 6= (x − a)k .
k=0
k!
Aim: Our aim is to find some conditions under which f can be given by its Taylor series
or equivalently remainder Rn (x) goes to zero as n tends to ∞.

38
Theorem 9.12. Taylor’s Theorem:
Let f be defined on (a, b) where a < 0 < b, and suppose the nth derivative f (n) exists on
(a, b). Then for each nonzero x in (a, b) there is y between 0 and x such that

f (n) (y) n
Rn (x) = x
n!
Remark 9.13. 1. For n = 1, Taylor’s Theorem is nothing but Mean Value Theorem.

2. The condition a < 0 < b is because we are taking the Taylor series of f around 0.

3. Note that y depends on n as well as x.

Corollary 9.14. Let f be defined on (a, b) where a < 0 < b. If all the derivatives f (n)
exist on (a, b) and are bounded by a single constant C, then

lim Rn (x) = 0 for all x ∈ (a, b).


n→∞

Proof : Observe that


f (n) (y) C
Rn (x) = xn ≤ |x|n for all n = 0, 1, 2, · · ·

n! n!
Hence we get limn→∞ Rn (x) = 0 for all x ∈ (a, b).

Proof : Fix x 6= 0. Define
Rn (x)n!
M= .
xn
We need to show that f (n) (y) = M for some y ∈ (0, x). Consider the function
n−1 (k)
X f (0) M tn
g(t) = tk + − f (t)
k=0
k! n!

Observe that
g(0) = 0 and g (k) (0) = 0 for k < n.
By the choice of M , we get
g(x) = 0.
By Rolle’s Theorem, there exists x1 ∈ (0, x) such that

g 0 (x1 ) = 0.

39
00
Again, since g (0) = 0, by Rolle’s Theorem, there exists x2 ∈ (0, x1 ) ⊂ (0, x) such that
00
g (x2 ) = 0.

Proceeding in this manner, we get xn ∈ (0, x) such that

g (n) (xn ) = 0.

Also, we have
g (n) (t) = M − f (n) (t).
This proves that f (n) (xn ) = M .


10 Integration
If f be a bounded function on a closed interval [a, b]. For S ⊂ [a, b], define

M (f, S) = sup{f (x) : x ∈ S} and m(f, S) = inf{f (x) : x ∈ S}.

A partition of [a, b] is any finite ordered subset P having the form

P = {a = t0 < t1 < ... < tn = b}.

The upper Darboux sum U (f, P ) of f with respect to P is


n
X
U (f, P ) = M (f, [tk−1 , tk ])(tk − tk−1 )
k=1

and the lower Darboux sum L(f, P ) is


n
X
L(f, P ) = m(f, [tk−1 , tk ](tk − tk−1 )
k=1

Claim:
m(f, [a, b])(b − a) ≤ L(f, P ) ≤ U (f, P ) ≤ M (f, [a, b])(b − a).
Proof : The middle inequality is obvious. The proofs of the first and third inequalities
are similar, so we will prove

m(f, [a, b])(b − a) ≤ L(f, P ).

40
Since m(f, [a, b]) ≤ m(f, [tk−1 , tk ] for all k, we have
n
X
L(f, P ) = m(f, [tk−1 , tk ](tk − tk−1 )
k=1
n
X
≤ L(f, P ) = m(f, [a, b])(tk − tk−1 )
k=1
Xn
= m(f, [a, b]) (tk − tk−1 )
k=1
= m(f, [a, b])(b − a).


The upper Darboux integral U (f ) of f over [a, b] is defined by

U (f ) = inf{U (P, f ) : P is a partition of [a, b]}

and lower Darboux integral L(f ) of f over [a, b] is defined by

L(f ) = inf{L(P, f ) : P is a partition of [a, b]}

Note that U (f ) < ∞ and L(f ) < ∞ (why?).

Definition 10.1. We say that f is integrable if U (f ) = L(f ). In this case we write


Z b Z b
f= f (x)dx = U (f ) = L(f ).
a a

Exercise: Consider the partition P = {−1, 0, 1} of [−1, 1]. Find L(f, P ) and U (f, P )
of the following function:

• f (x) = x

• f (x) = |x|

• f (x) = x2

Exercise: Show that the constant function f : [a, b] → R sending x 7→ c is integrable


and Z b
f = c(b − a).
a
Exercise: Show that the function f : [−1, 1] → R given by

0 if x 6= 0,
f (x) =
1 if x = 0,

41
is integrable and Z b
f = 0.
a
Exercise: Show that the function f : [0, 1] → R given by

0 if x ∈ Q,
f (x) =
1 if x ∈ / Q,
is not integrable.
Exercise: Consider
n
X
L(f, P ) = m(f, [tk−1 , tk ](tk − tk−1 ).
k=1

If f (x) ≤ k for all x then show that

L(f, P ) ≤ k(b − a).

Lemma 10.2. Let f be a bounded function on [a, b]. If P and Q are partitions of [a, b]
and P ⊂ Q, then
L(f, P ) ≤ L(f, Q) ≤ U (f, Q) ≤ U (f, P ).
Proof : The middle inequality is obvious. The proofs of the first and third inequalities
are similar, so we will prove
L(f, P ) ≤ L(f, Q).
Assume that Q has only one more point, say u, than P . General case follows by induction.
If
P = {a = t0 < t1 < · · · < tn = b}.
then
Q = {a = t0 < t1 < · · · < tk−1 < u < tk · · · < tn = b}.
for some k ∈ {1, 2, · · · , n}. The lower Darboux sums for P and Q are the same except
for the terms involving tk−1 or tk . In fact, their difference is

L(f, Q) − L(f, P )
= m(f, [tk−1 , u])(u − tk−1 ) + m(f, [u, tk ]).(tk − u) − m(f, [tk−1 , tk ]).(tk − tk−1 ).

To prove the claim, it suffices to show that this quantity is nonnegative. We have

m(f, [tk−1 , tk ]).(tk − tk−1 ) = m(f, [tk−1 , tk ]).(tk − u + u − tk−1 )


= m(f, [tk−1 , tk ]).(tk − u) + m(f, [tk−1 , tk ]).(u − tk−1 )
≤ m(f, [u, tk ]).(tk − u) + m(f, [tk−1 , u]).(u − tk−1 )

42
as m(f, [u, tk ]), m(f, [tk−1 , u]) ≥ m(f, [tk−1 , tk ]).


Lemma 10.3. If f is a bounded function on [a, b], and if P and Q are partitions of [a, b],
then L(f, P ) ≤ U (f, Q).

Proof : The set P ∪ Q is a partition of [a, b]. Since P ⊂ P ∪ Q and Q ⊂ P ∪ Q, we have

L(f, P ) ≤ L(f, P ∪ Q) ≤ U (f, P ∪ Q) ≤ U (f, Q). (10.1)

Lemma 10.4. If f is a bounded function on [a, b] then L(f ) ≤ U (f ).

Proof : Fix a partition P of [a, b]. Lemma 10.3 shows that L(f, P ) is a lowe bound for
the set
{U (f, Q) : Q is a partition of [a, b]}.
Therefore L(f, P ) must be less than or equal to the greatest lower bound [infimum!] of
this set. That is,
L(f, P ) ≤ U (f ).
This fact shows that U (f ) is an upper bound for the set

{U (f, Q) : Q is a partition of [a, b]}.

Hence
L(f ) ≤ U (f ).


Theorem 10.5. A bounded function f on [a, b] is integrable if and only if for each  > 0
there exists a partition P of [a, b] such that

U (f, P ) − L(f, P ) < .

Proof : Suppose first that f is integrable and consider  > 0. There exist partitions P1
and P2 of [a, b] such that
 
L(f, P1 ) > L(f ) − and U (f, P2 ) < U (f ) + .
2 2

43
For P = P1 ∪ P2 , we apply Lemma 10.3 to get

U (f, P ) − L(f, P ) ≤ U (f, P2 ) − L(f, P1 )


 
< U (f ) + − (L(f ) − )
2 2
= U (f ) − L(f ) + .
= as f is integrable.

Conversely, suppose that for each  > 0, there exists a partition P of [a, b] such that

U (f, P ) − L(f, P ) < .

Then we have

U (f ) ≤ U (f, P ) = (U (f, P ) − L(f, P )) + U (f, P ) <  + L(f, P ) ≤  + L(f )

Since it holds for all , we get


U (f ) ≤ L(f ).
Also, we have
U (f ) ≥ L(f ).
This implies
U (f ) = L(f ).


Theorem 10.6. A bounded function f on [a, b] is integrable if and only if for each  > 0
there exists δ > 0 such that

mesh(P ) < δ implies U (f, P ) − L(f, P ) < 

for all partitions P of [a, b].

Proof : The backward implication follows from Theorem 10.5. Tp prove the other part,
suppose that f is integrable on [a, b]. Fix  > 0. Choose a partition

P0 = {a = t0 < t1 < · · · < tn = b}.

such that

U (f, P0 ) − L(f, P0 ) < .
2

44
Let
B = sup |f (x)|.
x∈[a,b]

Take δ = 8mB
; m is the number of intervals comprising P0 . Now consider any partition

P = {a = t0 < t1 < · · · < tn = b}.

with mesh(P ) < δ. Let Q = P ∪ P0 . If Q has one more element, say u than P , then it
follows from the calculations done in Lemma 10.2 that

L(f, Q) − L(f, P )
≤ m(f, [tk−1 , u])(u − tk−1 ) + m(f, [u, tk ]).(tk − u) − m(f, [tk−1 , tk ]).(tk − tk−1 )
≤ B meshP − (−B) meshP = 2B meshP

Using induction, one can show that if Q has r more element than P , then

L(f, Q) − L(f, P ) ≤ 2rB meshP

So we have

L(f, Q) − L(f, P ) ≤ 2mB meshP


≤ 2mBδ

=
4
By Lemma 10.3, we have
L(f, P0 ) ≤ L(f, Q)
Hence

L(f, P0 ) − L(f, P ) <
4
similarly,

U (f, P ) − U (f, P0 ) <
4
So,

U (f, P ) − L(f, P ) < U (f, P0 ) − L(f, P0 ) + < .
2


45
10.1 Riemann Integration
Let f be a bounded function on a closed interval [a, b] and P be a partition of [a, b] of
the form
P = {a = t0 < t1 < ... < tn = b}.
A Riemann sum of f associated with P is a sum of the form
n
X
f (xk )(tk − tk−1 ).
k=1

where xk ∈ [tk−1 , tk ] for 1 ≤ k ≤ n. Here xk is choosen arbitrarily, so there are infinitely


many Riemann sums associated with a single function and partition.

Definition 10.7. A bounded function f is Riemann integrable on [a, b] if there exists a


number I with the following property. For each  > 0 there exists δ > 0 such that

|S − I| < 

for every Riemann sum S of f associated with a partition P having mesh(P ) < δ. The
number I is the Riemann integral of f on [a, b].

The following theorem says that the two notions of integrability are equivalent and
their values are also same.

Theorem 10.8. A bounded function f on [a, b] is Riemann integrable if and only if it


is integrable (Darboux), in which case the values of the integrals agree.

Proof : Suppose first that f is integrable on [a, b]. Let  > 0. By Theorem 10.6, choose
δ > 0 such that

mesh(P ) < δ implies U (f, P ) − L(f, P ) < 

for all partitions P of [a, b]. We need to show that


Z b
S − f 

a

for every Riemann sum


n
X
S= f (xk )(tk − tk−1 )
k=1

associated with a partition P having mesh(P ) < δ. Clearly we have

L(P, f ) ≤ S ≤ U (P, f ).

46
Moreover, by our choice of δ, we have
Z b
U (P, f ) < L(P, f ) +  ≤ L(f ) +  = f +
a
Z b
L(P, f ) > U (P, f ) −  ≥ U (f ) −  = f − .
a

Hence Z b Z b
f −≤S ≤ f + .
a a
This proves the first part of the claim.
Now suppose that f is Riemann integrable and let r be its value. Consider  > 0. Select
any partition
P0 = {a = t0 < t1 < · · · < tn = b}.
with mesh(P ) < δ and for each k = 1, 2, · · · , n select xk ∈ [tk−1 , tk ] so that

f (xk ) < m(f, [tk−1 , tk ]) + .

The Riemann sum S for this choice of xk ’s satisfies


n
X
S≤ m(f, [tk−1 , tk ]) +  = L(f, P ) + (b − a).
k=1

Also we have
S − r < .

Hence
L(f ) ≥ L(f, P ) ≥ S − (b − a) ≥ r −  − (b − a).
Since  is arbitrary, we have
L(f ) ≥ r.
A similar argument shows that
U (f ) ≤ r.
Since L(f ) ≤ U (f ), we have
Z b
L(f ) = U (f ) = f = r.
a


Exercise: Let f be a bounded function on [a, b] i.e. there exists B > 0 such that
|f (x)| ≤ B for all x ∈ [a, b].

47
1. Show that
U (f 2 , P ) − L(f 2 , P ) ≤ 2B[U (f, P ) − L(f, P )]
for all partitions P of [a, b].

Hint: Use f (x)2 − f (y)2 = [f (x) + f (y)][f (x) − f (y)].

2. Show that if f is integrable on [a, b], then f 2 also is integrable on [a, b].

Exercise: Let f and g be integrable functions on [a, b].

1. Show that f g is integrable on [a, b].

Hint: Use 4f g = (f + g)2 − (f − g)2 .

2. Show that max{f, g} and min{f, g} are integrable on [a, b].


f +g |f −g|
Hint: Use min{f, g} = 2
− 2
.

10.2 Properties:
Theorem 10.9. Every monotonic function f on [a, b] is integrable.

Proof : Assume f is monotonically increasing; the other case will follow by similar argu-
ment. Since
f (a) ≤ f (x) ≤ f (b) for all x ∈ [a, b],
it follows that f is a bounded function. bounded on [a, b]. Let  > 0 and

P = {a = t0 , t1 , · · · tn = b}

be a partition of [a, b] with mess less than f (b)−f (a)
. Then
n
X
U (f, P ) − L(f, P ) = (M (f, [tk − tk−1 ]) − m(f, [tk − tk−1 ]))(tk − tk−1 )
k=1
n
X
= (f (tk ) − f (tk−1 ))(tk − tk−1 )
k=1

Since mesh(P ) < f (b)−f (a)
, we have
n
X 
U (f, P ) − L(f, P ) < (f (tk ) − f (tk−1 ))
k=1
f (b) − f (a)

= (f (b) − f (a)) = .
f (b) − f (a)

48


Theorem 10.10. Every continuous function f on [a, b] is integrable.

Proof : Fix  > 0. Since f is uniformly continuous on [a, b], there exists δ > 0 such that

x, y ∈ [a, b] and |x − y| < δ ⇒ |f (x) − f (y)| <
b−a
Consider any partition
P = {a = t0 , t1 , · · · tn = b}
with mess less than δ. Since f assumes its maximum and minimum on each interval
[tk−1 , tk ], it follows that

M (f, [tk − tk−1 ]) − m(f, [tk − tk−1 ]) < .
b−a
Therefore we have
n
X 
U (f, P ) − L(f, P ) < (tk − tk−1 )(tk − tk−1 ) = .
k=1
b−a

Theorem 10.11. Linearity:


Let f and g be integrable functions on [a, b]. Then cf and f + g is integrable and we have
Z b Z b
cf = c f
a a
Z b Z b Z b
f +g = f+ g
a a a

Theorem 10.12. Monotonicity:


Let f and g be integrable functions on [a, b] and f (x) ≤ g(x) for all x ∈ [a, b]. Then one
has Z b Z b
f≤ g.
a a

Theorem 10.13. If f is integrable on [a, b], then |f | is integrable and


Z b Z b
f ≤ |f |.


a a

49
Theorem 10.14. Let f be a function defined on [a, b]. If a < c < b and f is integrable
on [a, c] and on [c, b], then f is integrable on [a, b] and
Z b Z c Z b
f= f+ f.
a a c

Proof : 
Exercise: If f is a continuous function on [a, b], then for at least one x in [a, b] we have
Z b
1
f (x0 ) = f.
b−a a
It is called Intermediate Value Theorem for Integrals.
Theorem 10.15. Fundamental Theorem of Calculus I:
If g is a continuous function on [a, b] that is differentiable on (a, b), and if g 0 is integrable
on [a, b], then
Z b
g 0 = g(b) − g(a).
a
Proof : Let  > O. By Theorem 10.5, there exists a partition P

P = {a = t0 , t1 , · · · · · · , tn−1 , tn = b}

of [a, b] such that


U (g 0 , P ) − L(g 0 , P ) < .
We apply the Mean Value Theorem to each interval [tk−1 , tk ] to obtain xk ∈ (tk−1 , tk ) for
which
(tk − tk−1 )g 0 (xk ) = g(tk ) − g(tk−1 ).
Hence n n
X X
g(b) − g(a) = g(tk ) − g(tk−1 ) = g 0 (xk )(tk − tk−1 ).
k=1 k=1
Therefore we have
L(g 0 , P ) ≤ g(b) − g(a) ≤ U (g 0 , P )
and Z b
0
L(g , P ) ≤ g 0 ≤ U (g 0 , P ).
a
Combining all these inequalities, we get
Z b
0 0 0
g − (g(b) − g(a)) ≤ U (g , P ) − L(g , P ) < .


a

Since  is arbitrary, we get the claim. 

50
xn+1
Example 10.16. If g(x) = n+1
then g 0 (x) = xn . So we have
Z 1
1
xn dx = g(1) − g(0) = .
0 n+1
Theorem 10.17. [Integration by parts]:
If u and v are continuous functions on [a, b] that are differentiable on (a, b), and if u0
and v 0 are integrable on [a, b], then
Z b Z b
0
uv+ v 0 u = u(b)v(b) − u(a)v(a)
a a

Proof : Let g = uv then g 0 = uv 0 + u0 v. Then g 0 is integrable. By first fundamental


theorem of calculus, we have
Z b Z b Z b
0 0
uv+ vu= g 0 = g(b) − g(a) = u(b)v(b) − u(a)v(a).
a a a

Theorem 10.18. Fundamental Theorem of Calculus II:


Let f be an integrable function on [a, b]. For x ∈ [a, b], define
Z x
F (x) = f
a

Then F is continuous on [a, b]. If f is continuous at c in (a, b), then F is differentiable


at c and
F 0 (c) = f (c).

Proof : Choose M such that |f (x)| ≤ M for all x ∈ [a, b]. Fix  > 0. For x, y ∈ [a, b]
such that |x − y| < B and y > x, we have
Z y Z y Z y
|F (y) − F (x)| ≤ f (t)dt ≤ |f (t)|dt ≤ Bdt ≤ B(y − x) < .

x x x

This proves that F is uniformly continuous on [a, b].


Suppose f is continuous at c. Note that
Z x
F (x) − F (c) 1
= f (t)dt
x−c x−c c
and Z x
1
f (c) = f (c)dt.
x−c c

51
Therefore we have
F (x) − F (c) 1 Z x 1
Z x
− f (c) = (f (t) − f (c))dt ≤ |f (t) − f (c)|dt.

x−c x−c c |x − c| c

Since f is continuous at c, there exists δ > 0 such that

t ∈ (a, b) and |t − c| < δ imply |f (t) − f (c)| < .

Hence for t ∈ (a, b) and |t − c| < δ, we have


F (x) − F (c)
− f (c) ≤ .

x−c

This proves that


F (x) − F (c)
lim = f (c).
x→c x−c


Theorem 10.19. [Change of variables]:


Let u be a differentiable function on an open interval J such that u0 is continuous, and
let I be an open interval such that u(x) ∈ I for all x ∈ J. If f is continuous on I, then
f ◦ u is continuous on J and
Z b Z u(b)
0
(f ◦ u)(x)u (x)dx = f (u)du.
a u(a)

for a, b ∈ J.
Proof : The continuity of f ◦ u follows from the fact that composition of two continuous
functions are continuous. Fix c ∈ I and let
Z u(x)
F (u(x)) = f (t)dt.
c
0
Then F (u(x)) = f (u(x)) for all x ∈ J. Let g = F ◦ u. By chain rule, we have

g 0 (x) = F 0 (u(x))u0 (x) = f (u(x))u0 (x).

Hence we get
Z b Z b
0
(f ◦ u)(x)u (x)dx = g 0 (x)dx = g(b) − g(a) = F (u(b)) − F (u(a))
a
Za u(b) Z u(a)
= f (t)dt − f (t)dt
c c
Z u(b)
= f (t)dt.
u(a)

52
11 Improper Integration

References
[1] Joseph C. Várilly. Dirac Operators and Spectral Geometry. http://toknotes.
mimuw.edu.pl/sem3/files/Varilly_dosg.pdf.

53

You might also like